Insufficient rain can cause crops to falter and agricultural prices to rise. Records indicate that during a certain n...

bcross on June 10, 2020

Why not E?

I got the correct answer, but I was stuck between D and E. Why would E be incorrect?

Reply
Create a free account to read and take part in forum discussions.

Already have an account? log in

shunhe on June 14, 2020

Hi @bcross,

Thanks for the question! So let’s take a look at what (E) is telling us. When it says that the argument takes for granted that conditions necessary for an increase in agricultural prices are also sufficient for such increases, what it’s saying is that the argument does a mistaken reversal. If given X —> Y, the argument concludes X from Y, instead of the other way around. That’s not quite what happens here, since there’s not really conditional logic happening here. What’s going on here has more to do with causation than errors in conditional logic.

Hope this helps! Feel free to ask any other questions that you might have.